Đến nội dung

Ngockhanh99k48 nội dung

Có 114 mục bởi Ngockhanh99k48 (Tìm giới hạn từ 26-05-2020)



Sắp theo                Sắp xếp  

#645014 Tìm max,min : P=$\frac{6-2(x-1)(y-1)}{(x-1)^{2...

Đã gửi bởi Ngockhanh99k48 on 15-07-2016 - 01:10 trong Hàm số - Đạo hàm

Đặt $a=x-1, b=y-1$, theo giả thiết thì $a^2+b^2=\frac{1}{2}$. Và P=12-4ab. Do $(a\sqrt{2})^2+(b\sqrt{2})^2=1$ nên tồn tại $\alpha \in [0; 2\pi]$ sao cho $a=\frac{\cos\alpha}{\sqrt{2}}, b=\frac{\sin\alpha}{\sqrt{2}}$. Khi đó: $P=12-2\sin\alpha.\cos\alpha=12-\sin 2\alpha$. Đến đây chắc bạn làm nốt được rồi đúng không? :)



#645015 CM: AM $\perp$ EF.

Đã gửi bởi Ngockhanh99k48 on 15-07-2016 - 01:21 trong Hình học

Trên tia đối của tia $BC$ lấy $G$ sao cho: $DE=BG$. Do đó $\triangle ADE =\triangle ABG$(c-g-c). Suy ra $AE=AG$ và $AE \perp AG$. Do đó $AF$ là phân giác góc $\widehat{EAF}$. Từ đó suy ra $\triangle AEF =\triangle AGF$. Khi đó $FA$ là phân giác góc $\widehat{EFG}$. $AM$ cắt $EF$ tại $K$ thì $\triangle AFK =\triangle AFB$(g-c-g). Từ đó $AM \perp EF$ tại $K$.



#645017 Chứng minh $\frac{a}{ca+1}+\frac{b...

Đã gửi bởi Ngockhanh99k48 on 15-07-2016 - 01:37 trong Bất đẳng thức và cực trị

Đoạn trên ta có thể làm tiếp như sau: $ab+bc+ca \leq a^2+b^2+c^2$ và $(a^2+b^2+c^2)^3 \geq \frac{(a+b+c)^2}{3}.(ab+bc+ca)^2 \geq (a+b+c)^2.abc(a+b+c)=(a+b+c)^3$ $\Rightarrow$ $a+b+c \leq a^2+b^2+c^2$.



#645019 So sánh $x=\frac{a}{b}; y=\frac{c}{d}; z=\frac{m}{n}; m=...

Đã gửi bởi Ngockhanh99k48 on 15-07-2016 - 01:43 trong Số học

Nếu $x \leq y$ thì ta sẽ có $x \leq z \leq y$. Tương tự với $x \geq y$



#645021 $\frac{a^{2}}{b+c}+\frac{b^...

Đã gửi bởi Ngockhanh99k48 on 15-07-2016 - 02:13 trong Bất đẳng thức và cực trị

Sử dụng BĐT Holder ta có:
$(\sum_{cyc}\frac{a^2}{b+c})^2(\sum_{cyc}a^2(b+c)^2) \geq (a^2+b^2+c^2)^3$. Từ đó ta đi chứng minh $\frac{(a^2+b^2+c^2)^6}{(\sum_{cyc}a^2(b+c)^2)^2} \geq \frac{27(a^4+b^4+c^4)}{16}$. Mặt khác ta có $\sum_{cyc}a^2(b+c)^2 = a^2b^2+b^2c^2+c^2a^2 +(ab+bc+ca)^2 \leq 4(a^2b^2+b^2c^2+c^2a^2)$. Từ đó ta chỉ cần chứng minh $(a^2+b^2+c^2)^6 \geq 27(a^4+b^4+c^4)(a^2b^2+b^2c^2+c^2a^2)^2$. BĐT trên đúng theo AM-GM nên ta có đpcm.



#645039 So sánh $x=\frac{a}{b}; y=\frac{c}{d}; z=\frac{m}{n}; m=...

Đã gửi bởi Ngockhanh99k48 on 15-07-2016 - 10:28 trong Số học

Nếu $x < y$ thì $\frac{a}{c} < \frac{b}{d}$ $\Rightarrow$ $\frac{a+c}{c} < \frac{b+d}{d}$ $\Rightarrow$ $\frac{a+c}{b+d} < \frac{c}{d}$ $\Rightarrow$ $z <y$. Tương tự ta sẽ chứng minh được $x <z$.



#645356 CMR: $(a+b)^5 \geg 32ab$

Đã gửi bởi Ngockhanh99k48 on 17-07-2016 - 23:42 trong Bất đẳng thức và cực trị

Đề bài sai nhé, thử với $a=0,1$ và $b=0,2$.



#645613 Chứng minh: $TP\parallel HQ.$

Đã gửi bởi Ngockhanh99k48 on 20-07-2016 - 09:16 trong Hình học

$BO$ cắt $(O)$ tại điểm thứ hai $S$. Khi đó ta cũng có $AHCS$ là hình bình hành (có các cạnh đối song song). Suy ra $H, Q, S$ thẳng hàng. Ta có $\triangle BDT \sim \triangle BAP (gg)$ và $\triangle BHD \sim \triangle BSA (gg)$. Từ đó ta có hệ thức $\frac{BT}{BP}=\frac{BD}{BA}=\frac{BH}{BS}$. Do đó $TP \parallel HQ$.



#645617 Hệ phương trình

Đã gửi bởi Ngockhanh99k48 on 20-07-2016 - 09:48 trong Phương trình, hệ phương trình và bất phương trình

1) Phương trình thứ hai được viết lại thành $2(x+1)^3+3(x+1)^2y+4y^3=0$. Giải ra ta có $x+1=-2y$. Thay vào phương trình thứ nhất là xong.



#645620 Hệ phương trình

Đã gửi bởi Ngockhanh99k48 on 20-07-2016 - 10:06 trong Phương trình, hệ phương trình và bất phương trình

3) Phương trình thứ hai được viết lại thành $(3xy-1)(2x^2-2xy+y^2+1)=0$. Do đó $3xy=1$. Thế vào phương trình thứ nhất là xong.



#645622 Hệ phương trình

Đã gửi bởi Ngockhanh99k48 on 20-07-2016 - 10:18 trong Phương trình, hệ phương trình và bất phương trình

4) Phương trình thứ hai được viết lại thành $(xy-x-1)^2=0$. Do đó $y=1+\frac{1}{x}$. Suy ra $y^3=1+\frac{1}{x^3}+3.1.\frac{1}{x}(1+\frac{1}{x})=1+\frac{1}{x^3}+\frac{3y}{x}(*)$.
Phương trình thứ nhất được viết lại thành $y^3+3(\frac{y}{x})^2-7(\frac{y}{x})^3=\frac{1}{x^3}$. Thế $(*)$ vào ta được phương trình bậc 3 ẩn $t=\frac{y}{x}$. Tìm được $y$ theo $x$ ta dễ dàng suy ra nghiệm và thử lại.



#645625 Hệ phương trình

Đã gửi bởi Ngockhanh99k48 on 20-07-2016 - 10:22 trong Phương trình, hệ phương trình và bất phương trình

mình biết là nó có nhân tử như vậy như bị rối ở chỗ phân tích pt 2 thành pt tích ấy. Bạn phân tích giúp mình đc ko?

$(x+1+2y)[2(x+1)^2-y(x+1)+2y^2]=0$



#646492 Giải pt $2x^{2}+5x-1=7\sqrt{x^{3}-1}...

Đã gửi bởi Ngockhanh99k48 on 25-07-2016 - 21:58 trong Bất đẳng thức và cực trị

Viết lại như sau:
$2(x^2+x+1)+3(x-1)=7\sqrt{(x-1)(x^2+x+1)}$



#646495 Chứng minh $QB=PC$

Đã gửi bởi Ngockhanh99k48 on 25-07-2016 - 22:12 trong Hình học

Thay như bạn Le Nguyen Tan Kiet là đúng.
Lời giải: kẻ tiếp tuyến $SH$ của $(O)$. Khi đó tứ giác DBHC là tứ giác điều hòa nên $A(BCMH)=A(BCDH)=-1$. Do $M$ là trung điểm $BC$ nên $AH \parallel BC$. Cũng do tứ giác $DFHE$ điều hòa nên $A(MHQP)=A(DHFE)=-1$. Do $AH \parallel PQ$ nên $M$ là trung điểm $PQ$.



#646707 VMF's Marathon Hình học Olympic

Đã gửi bởi Ngockhanh99k48 on 27-07-2016 - 13:29 trong Hình học

$\boxed{\text{Lời giải bài toán 85}}$

$AP, BP, CP$ cắt $(ABC)$ thứ tự tại điểm thứ hai $X, Y, Z$. Đương nhiên dễ thấy rằng $\triangle XYZ$ và $\triangle DEF$ có các cạnh tương ứng song song và có tâm vị tự là $P$. Do đó giả sử gọi $L$ là điểm Lemoine của $\triangle XYZ$ thì điểm Lemoine của $\triangle DEF$ thuộc $PL$. Do $A, B, C, P$ cố định nên $X, Y, Z$ cố định hay $L$ cố định nên $PL$ cố định, ta có đpcm

P/s: Thầy có thể tiếp tục giúp em được không ạ?




#646748 VMF's Marathon Hình học Olympic

Đã gửi bởi Ngockhanh99k48 on 27-07-2016 - 16:35 trong Hình học

Bài toán 85': Cho $\triangle ABC$ nội tiếp đường tròn $(O)$ với điểm $P$ bất kì, $AP, BP, CP$ thứ tự cắt $(ABC)$ tại điểm thứ hai $D, E, F$. Gọi $L, L'$ thứ tự là điểm Lemoine của $\triangle ABC, \triangle DEF$. Khi đó $P, L, L'$ thẳng hàng. 

Chứng minh: $AL$ cắt $(O)$ tại điểm thứ hai $A'$. $A'P$ cắt $(O)$ tại điểm thứ hai $D'$. $AA'$ là đường đối trung của tam giác $\triangle ABC$ nên $(AA'BC)=-1$. Ta biết rằng phép nghịch đảo bảo toàn tỉ số kép, mặt khác $AD, BE, CF, A'D'$ đồng quy tại $P$, nên $(DD'EF)=(AA'BC)=-1$, do đó $DD'$ đi qua $L'$. Định nghĩa tương tự với $B', E'$ thì $EE'$ đi qua điểm $L'$. Gọi $K$ là giao điểm của $AE'$ và $BD'$

Bây giờ áp dụng định lí Pascal cho bộ 6 điểm đồng viên $\begin{pmatrix} A & B' & D' \\  B & A' & E' \end{pmatrix}$ ta có $L, P, K$ thẳng hàng, lại áp dụng định lí Pascal cho bộ 6 điểm đồng viên $\begin{pmatrix} A & D' & E \\  B & E' & D \end{pmatrix}$ ta có $K, P, L'$ thẳng hàng. Ta có đpcm.

$\boxed{\text{Lời giải bài toán 86}}$ $AI$ cắt $CD$ tại $F$. Một tính chất đặc trưng của tứ giác ngoại tiếp đó là $\widehat{AID}+\widehat{BIC} = 180^{\circ}$. Do đó $\widehat{FID}=\widehat{CIB}$, suy ra hai góc $\widehat{CIF}$ và $\widehat{BID}$ có chung đường phân giác trong. Từ đó ta có đpcm. 




#646799 chứng minh: $\frac{(x+y)(x+z)}{4x}+\frac...

Đã gửi bởi Ngockhanh99k48 on 27-07-2016 - 20:55 trong Bất đẳng thức và cực trị

Cho các số dương $x,y,z$. Chứng minh :

$$\frac{(x+y)(x+z)}{4x}+\frac{(y+x)(y+z)}{4y}+\frac{(x+z)(z+y)}{4z}\geq x+y+z$$

$\Leftrightarrow$ $\frac{x(x+y+z)+yz}{x}+\frac{y(x+y+z)+xz}{y}+\frac{z(x+y+z)+xy}{z} \geq 4(x+y+z)$ $\Leftrightarrow$ $\frac{yz}{x}+\frac{xz}{y}+\frac{xy}{z} \geq x+y+z$. Sử dụng AM-GM ta có: $\frac{xy}{z}+\frac{yz}{x} \geq 2y$. Tương tự ta có đpcm. 




#647329 $\frac{a^2}{b^2} + \frac{b^2}{a^2} \geq \frac{a}{b}...

Đã gửi bởi Ngockhanh99k48 on 31-07-2016 - 13:27 trong Bất đẳng thức và cực trị

$\Leftrightarrow$ $a^4+b^4 \geq ab(a^2+b^2)$ $\Leftrightarrow$ $(a-b)^2[(a+b)^2-ab] \geq 0$



#647857 ​$\frac{a}{a^2+2}+\frac{b}{...

Đã gửi bởi Ngockhanh99k48 on 03-08-2016 - 23:52 trong Bất đẳng thức và cực trị

Xử lí nhẹ :) $a^2+2 \geq 2a+1$. Do đó ta cần chứng minh $\frac{a}{2a+1}+\frac{b}{2b+1}+\frac{c}{2c+1} \leq 1$ $\Leftrightarrow$ $\frac{1}{2a+1}+\frac{1}{2b+1}+\frac{1}{2c+1} \geq 1$.
Giả thiết $abc=1$ nên tồn tại $x, y, z >0$ thỏa $a=\frac{x}{y}, b=\frac{y}{z}, c=\frac{z}{x}$. Từ đó, sử dụng Cauchy-schwarz ta có $\frac{1}{2a+1}+\frac{1}{2b+1}+\frac{1}{2c+1}=\frac{y}{2x+y}+\frac{z}{2y+z}+\frac{x}{2z+x} \geq \frac{(x+y+z)^2}{y(2x+y)+z(2y+z)+x(2z+x)}=1$.



#647932 Chứng minh rằng: $2(x+y+z)-xyz\le 10$.

Đã gửi bởi Ngockhanh99k48 on 04-08-2016 - 16:30 trong Bất đẳng thức và cực trị

Lời giải bài 2: Đặt $x=3a, y=5b, z=2c$ với $a, b, c >0$. Theo giả thiết thì ta có: $3a+5b+7c=15abc$.
Ta có $P=6a+5b+4c$
Sử dụng AM-GM ta có: $(6a+5b+4c)^2(3a+5b+7c) \geq 225\sqrt[15]{a^{12}b^{10}c^8}. 15\sqrt[15]{a^3b^5c^7}=15^3abc$. Suy ra $P \geq 15$. Dấu bằng xảy ra khi: $x=3, y=4, z=2$



#647939 Chứng minh rằng: $2(x+y+z)-xyz\le 10$.

Đã gửi bởi Ngockhanh99k48 on 04-08-2016 - 16:56 trong Bất đẳng thức và cực trị

Bài 1: Theo mình không cần $xyz \leq 0$ đâu. Không mất tổng quát, giả sử $z= \max \{x, y, z \}$. Khi đó $18 = 2x^2+2y^2+2z^2 \geq 3(x^2+y^2) \geq 6xy$, suy ra $xy \leq 3$.
Áp dụng BĐT Cauchy-Schwarz ta có $[2(x+y+z)-xyz ] ^2 = [2(x+y)+z(2-xy)]^2$ $\leq [2^2+(2-xy)^2 ][(x+y)^2+z^2 ] = (9+2xy)(x^2y^2-4xy+8) = 100 + (2xy-7)(xy+2)^2 \leq 100$. Ta có đpcm. Dấu bằng xảy ra khi $x=z=2, y=-1$ và hoán vị.
P/s: bạn có thể xem thêm VNTST 2002



#647955 Tìm GTNN của $P=(a+b)(a+c)$

Đã gửi bởi Ngockhanh99k48 on 04-08-2016 - 18:18 trong Bất đẳng thức và cực trị

Theo giả thiết thì $abc(a+b+c)=1$.
Sử dụng AM-GM: $P=(a+b)(a+c)=a(a+b+c)+bc \geq 2\sqrt{abc(a+b+c)}=2$



#648000 $x+y+z+\frac{1}{x+y+z}\geq \frac{10}{3}$

Đã gửi bởi Ngockhanh99k48 on 04-08-2016 - 23:26 trong Bất đẳng thức và cực trị

Bài 2: Sử dụng Cauchy-Schwarz ta có:
$\sqrt{(x+y)(y+z)(x+z)}.\sqrt{x+y} \geq \(x+y)\sqrt{(x+z)(y+z)} \geq (x+y)(\sqrt{xy}+z) = z(x+y) +(x+y)\sqrt{xy} \geq z(x+y)+2xy$. Lập tương tự các công thức ta có đpcm.



#648201 Tìm GTNN của biểu thức: $P= \frac{a+3c}{a+b}+\frac{c+3b}{b+c}+...

Đã gửi bởi Ngockhanh99k48 on 06-08-2016 - 11:48 trong Bất đẳng thức và cực trị

Đề đúng hình như là: $P= \frac{a+3c}{a+b}+\frac{c+3a}{b+c}+\frac{4b}{c+a}$.

Đặt $a+b=z, b+c=x, c+a=y$ với $x, y,z >0$. Khi đó $a=\frac{y+z-x}{2}, b=\frac{x+z-y}{2}, c=\frac{x+y-z}{2}$.
Thay vào biểu thức ta có:
$P=\frac{x+2y-z}{z}+\frac{z+2y-x}{x}+\frac{2x+2z-2y}{y} =(\frac{x}{z}+\frac{z}{x})+2y(\frac{1}{z}+\frac{1}{x})+ \frac{2(x+z)}{y} -4$.
Sử dụng AM-GM ta có:
$P \geq 2 + \frac{8y}{x+z}+ \frac{2(x+z)}{y} -4 \geq 2+8-4 =6$. Dấu bằng xảy ra khi $x=y=z$ $\Leftrightarrow$ $a=b=c$.



#648202 Chứng minh rằng $\sum \frac{2a^2-bc}{b^2-bc+c^2...

Đã gửi bởi Ngockhanh99k48 on 06-08-2016 - 11:59 trong Bất đẳng thức - Cực trị

Cho a,b,c là các sô thực không âm. Chứng minh rằng

$\sum \frac{2a^2-bc}{b^2-bc+c^2}\geq 3$

Bất đẳng thức cần chứng minh tương đương với:
$\sum_{cyc} (\frac{2a^2-bc}{b^2-bc-c^2}+1) \geq 6$ $\Leftrightarrow$ $\sum_{cyc} \frac{2a^2+(b-c)^2}{b^2-bc+c^2} \geq 6$.
Sử dụng Bất đẳng thức Cauchy-Schwarz ta có:
$(\sum_{cyc}\frac{2a^2+(b-c)^2}{b^2-bc-c^2})(\sum_{cyc} [2a^2+(b-c)^2](b^2-bc+c^2)) \geq 4(2 \sum_{cyc} a^2 -\sum_{cyc} ab)^2$.
Do đó ta cần chứng minh:
$2(2 \sum_{cyc} a^2- \sum_{cyc} ab)^2 \geq 3 \sum_{cyc} [2a^2+(b-c)^2][b^2-bc+c^2]$ $\Leftrightarrow$ $2 \sum_{cyc}a^4+2abc\sum_{cyc}a + \sum_{cyc} ab(a^2+b^2) \geq 6 \sum_{cyc} a^2b^2$.
Sử dụng BĐT Schur bậc 4 và AM-GM ta có đpcm. Dấu bằng xảy ra khi $a=b=c$ hoặc $a=b, c=0$ và các hoán vị.